How to solve this Diophantine equation?












4












$begingroup$


Can anyone say how one can find solutions to the Diophantine equation $$x^3+y^4=z^2$$ in General? Only a few triples of numbers have been found, and most likely this equation has infinitely many solutions.



Examples of triples: $(6,5,29),(2,1,3),(9,6,45)$...










share|cite|improve this question











$endgroup$

















    4












    $begingroup$


    Can anyone say how one can find solutions to the Diophantine equation $$x^3+y^4=z^2$$ in General? Only a few triples of numbers have been found, and most likely this equation has infinitely many solutions.



    Examples of triples: $(6,5,29),(2,1,3),(9,6,45)$...










    share|cite|improve this question











    $endgroup$















      4












      4








      4


      1



      $begingroup$


      Can anyone say how one can find solutions to the Diophantine equation $$x^3+y^4=z^2$$ in General? Only a few triples of numbers have been found, and most likely this equation has infinitely many solutions.



      Examples of triples: $(6,5,29),(2,1,3),(9,6,45)$...










      share|cite|improve this question











      $endgroup$




      Can anyone say how one can find solutions to the Diophantine equation $$x^3+y^4=z^2$$ in General? Only a few triples of numbers have been found, and most likely this equation has infinitely many solutions.



      Examples of triples: $(6,5,29),(2,1,3),(9,6,45)$...







      calculus diophantine-equations natural-numbers






      share|cite|improve this question















      share|cite|improve this question













      share|cite|improve this question




      share|cite|improve this question








      edited Dec 29 '18 at 12:45









      Martín Vacas Vignolo

      3,810623




      3,810623










      asked Dec 29 '18 at 12:43









      Yan DashkowYan Dashkow

      241




      241






















          5 Answers
          5






          active

          oldest

          votes


















          4












          $begingroup$

          This is a case of the generalized Fermat equation
          $$
          x^p+y^q=z^r.
          $$

          For $(p,q,r)=(3,4,2)$ we have $frac{1}{p}+frac{1}{q}+frac{1}{r}>1$, which is the spherical case. Here we have infinitely many integer solutions for this triple. The solutions are given by a finite set of polynomial parametrisations of the equation, see the following paper:



          F. Beukers, The diophantine equation $Ax^p + By^q = Cz^r$, Duke Math.J. 91(1998), 61-88.



          Further Reference: The generalized Fermat equation.






          share|cite|improve this answer











          $endgroup$





















            3












            $begingroup$

            Here is one simple parameterization. We have,



            $$x^4 +(y^2-1)^3 = (y^3+3y)^2$$



            given the Pell equation $x^2-3y^2 =1$.






            share|cite|improve this answer











            $endgroup$













            • $begingroup$
              @AlexD $r=2$ in this question.
              $endgroup$
              – Dietrich Burde
              Dec 29 '18 at 23:34










            • $begingroup$
              Thanks, @DietrichBurde. Let me try again: A solution to the problem here is a tuple of 3 numbers. But the parameterization here only relates $x$ and $y$. What about $z$?
              $endgroup$
              – Alex D
              Dec 30 '18 at 11:21










            • $begingroup$
              @AlexD We have $X^4+Y^3=Z^2$ and $X$,$Y$,$Z$ depend on $x$ and $y$ only. So no $z$ in here, only two parameters $x,y$ here for an equation in three variables $X,Y,Z$.
              $endgroup$
              – Dietrich Burde
              Dec 30 '18 at 11:47












            • $begingroup$
              Well, @DietrichBurde, you can find out more, but how can you find a general formula (solve this equation in general form), expressed in arbitrary numbers (for example) m, n, k? For example, find the top three numbers (6, 5, 29)?
              $endgroup$
              – Yan Dashkow
              Dec 30 '18 at 11:55










            • $begingroup$
              @YanDashkow We find them using one of the parametrizations, see Beukers article.
              $endgroup$
              – Dietrich Burde
              Dec 30 '18 at 13:31





















            0












            $begingroup$

            Above equation shown below has parameterization:



            $x^3+y^4=z^2$



            The below parameterization has no restriction such as the



            Pell equation condition demonstrated by Tito Piezas.



            $x=(p)^2(-q)^3$



            $y=(p)(q)^2(k-1)$



            $z=(p)^2(q)^4(2k-3)$



            where, $p=(k-2)$ and $q=(k^2-2)$



            For $k=3$ we get :
            $(-343)^3+(98)^4=(7203)^2$






            share|cite|improve this answer









            $endgroup$













            • $begingroup$
              I have a question: through which k you can find three solutions (x, y, z) — (6,5,29)?
              $endgroup$
              – Yan Dashkow
              Dec 30 '18 at 10:55





















            0












            $begingroup$

            "OP" asked for parametric solution for $(x,y,z)=(6,5,29)$ in $x^3+y^4=z^2$



            Solution is:



            $x=3p^3(8k^2-40k+50)$



            $y=p^2(20k^2-104k+135)$



            $z=p^4(2k-5)^2(116k^2-540k+621)$



            Where, $p=(4k^2-27)$



            For $k=(13/5)$, we get after removing common factors:



            $6^3+5^4=29^2$






            share|cite|improve this answer









            $endgroup$













            • $begingroup$
              Wait, why can't k be whole?
              $endgroup$
              – Yan Dashkow
              Dec 30 '18 at 19:30



















            0












            $begingroup$

            "OP" enquired about integer coefficent's for the parametric



            solution for the equation $(x^2+y^4=z^2)$. "OP" just needs



            to substitute $k=(m/n)$ in the parametrization & the resulting



            solution after removing common factors is given below.



            $x=6(u^3)(v^2)$



            $y=(u^2)(v)(10m-27n)$



            $z=(u^4)(v^2)(116m^2-540mn+621n^2)$



            And $u=(4m^2-27n^2)$ & $v=(2m-5n)$



            For $(m,n)=(13,5)$ we get:



            $6^3+5^4=29^2$






            share|cite|improve this answer









            $endgroup$













              Your Answer





              StackExchange.ifUsing("editor", function () {
              return StackExchange.using("mathjaxEditing", function () {
              StackExchange.MarkdownEditor.creationCallbacks.add(function (editor, postfix) {
              StackExchange.mathjaxEditing.prepareWmdForMathJax(editor, postfix, [["$", "$"], ["\\(","\\)"]]);
              });
              });
              }, "mathjax-editing");

              StackExchange.ready(function() {
              var channelOptions = {
              tags: "".split(" "),
              id: "69"
              };
              initTagRenderer("".split(" "), "".split(" "), channelOptions);

              StackExchange.using("externalEditor", function() {
              // Have to fire editor after snippets, if snippets enabled
              if (StackExchange.settings.snippets.snippetsEnabled) {
              StackExchange.using("snippets", function() {
              createEditor();
              });
              }
              else {
              createEditor();
              }
              });

              function createEditor() {
              StackExchange.prepareEditor({
              heartbeatType: 'answer',
              autoActivateHeartbeat: false,
              convertImagesToLinks: true,
              noModals: true,
              showLowRepImageUploadWarning: true,
              reputationToPostImages: 10,
              bindNavPrevention: true,
              postfix: "",
              imageUploader: {
              brandingHtml: "Powered by u003ca class="icon-imgur-white" href="https://imgur.com/"u003eu003c/au003e",
              contentPolicyHtml: "User contributions licensed under u003ca href="https://creativecommons.org/licenses/by-sa/3.0/"u003ecc by-sa 3.0 with attribution requiredu003c/au003e u003ca href="https://stackoverflow.com/legal/content-policy"u003e(content policy)u003c/au003e",
              allowUrls: true
              },
              noCode: true, onDemand: true,
              discardSelector: ".discard-answer"
              ,immediatelyShowMarkdownHelp:true
              });


              }
              });














              draft saved

              draft discarded


















              StackExchange.ready(
              function () {
              StackExchange.openid.initPostLogin('.new-post-login', 'https%3a%2f%2fmath.stackexchange.com%2fquestions%2f3055812%2fhow-to-solve-this-diophantine-equation%23new-answer', 'question_page');
              }
              );

              Post as a guest















              Required, but never shown

























              5 Answers
              5






              active

              oldest

              votes








              5 Answers
              5






              active

              oldest

              votes









              active

              oldest

              votes






              active

              oldest

              votes









              4












              $begingroup$

              This is a case of the generalized Fermat equation
              $$
              x^p+y^q=z^r.
              $$

              For $(p,q,r)=(3,4,2)$ we have $frac{1}{p}+frac{1}{q}+frac{1}{r}>1$, which is the spherical case. Here we have infinitely many integer solutions for this triple. The solutions are given by a finite set of polynomial parametrisations of the equation, see the following paper:



              F. Beukers, The diophantine equation $Ax^p + By^q = Cz^r$, Duke Math.J. 91(1998), 61-88.



              Further Reference: The generalized Fermat equation.






              share|cite|improve this answer











              $endgroup$


















                4












                $begingroup$

                This is a case of the generalized Fermat equation
                $$
                x^p+y^q=z^r.
                $$

                For $(p,q,r)=(3,4,2)$ we have $frac{1}{p}+frac{1}{q}+frac{1}{r}>1$, which is the spherical case. Here we have infinitely many integer solutions for this triple. The solutions are given by a finite set of polynomial parametrisations of the equation, see the following paper:



                F. Beukers, The diophantine equation $Ax^p + By^q = Cz^r$, Duke Math.J. 91(1998), 61-88.



                Further Reference: The generalized Fermat equation.






                share|cite|improve this answer











                $endgroup$
















                  4












                  4








                  4





                  $begingroup$

                  This is a case of the generalized Fermat equation
                  $$
                  x^p+y^q=z^r.
                  $$

                  For $(p,q,r)=(3,4,2)$ we have $frac{1}{p}+frac{1}{q}+frac{1}{r}>1$, which is the spherical case. Here we have infinitely many integer solutions for this triple. The solutions are given by a finite set of polynomial parametrisations of the equation, see the following paper:



                  F. Beukers, The diophantine equation $Ax^p + By^q = Cz^r$, Duke Math.J. 91(1998), 61-88.



                  Further Reference: The generalized Fermat equation.






                  share|cite|improve this answer











                  $endgroup$



                  This is a case of the generalized Fermat equation
                  $$
                  x^p+y^q=z^r.
                  $$

                  For $(p,q,r)=(3,4,2)$ we have $frac{1}{p}+frac{1}{q}+frac{1}{r}>1$, which is the spherical case. Here we have infinitely many integer solutions for this triple. The solutions are given by a finite set of polynomial parametrisations of the equation, see the following paper:



                  F. Beukers, The diophantine equation $Ax^p + By^q = Cz^r$, Duke Math.J. 91(1998), 61-88.



                  Further Reference: The generalized Fermat equation.







                  share|cite|improve this answer














                  share|cite|improve this answer



                  share|cite|improve this answer








                  edited Dec 29 '18 at 13:46

























                  answered Dec 29 '18 at 13:14









                  Dietrich BurdeDietrich Burde

                  78.2k64386




                  78.2k64386























                      3












                      $begingroup$

                      Here is one simple parameterization. We have,



                      $$x^4 +(y^2-1)^3 = (y^3+3y)^2$$



                      given the Pell equation $x^2-3y^2 =1$.






                      share|cite|improve this answer











                      $endgroup$













                      • $begingroup$
                        @AlexD $r=2$ in this question.
                        $endgroup$
                        – Dietrich Burde
                        Dec 29 '18 at 23:34










                      • $begingroup$
                        Thanks, @DietrichBurde. Let me try again: A solution to the problem here is a tuple of 3 numbers. But the parameterization here only relates $x$ and $y$. What about $z$?
                        $endgroup$
                        – Alex D
                        Dec 30 '18 at 11:21










                      • $begingroup$
                        @AlexD We have $X^4+Y^3=Z^2$ and $X$,$Y$,$Z$ depend on $x$ and $y$ only. So no $z$ in here, only two parameters $x,y$ here for an equation in three variables $X,Y,Z$.
                        $endgroup$
                        – Dietrich Burde
                        Dec 30 '18 at 11:47












                      • $begingroup$
                        Well, @DietrichBurde, you can find out more, but how can you find a general formula (solve this equation in general form), expressed in arbitrary numbers (for example) m, n, k? For example, find the top three numbers (6, 5, 29)?
                        $endgroup$
                        – Yan Dashkow
                        Dec 30 '18 at 11:55










                      • $begingroup$
                        @YanDashkow We find them using one of the parametrizations, see Beukers article.
                        $endgroup$
                        – Dietrich Burde
                        Dec 30 '18 at 13:31


















                      3












                      $begingroup$

                      Here is one simple parameterization. We have,



                      $$x^4 +(y^2-1)^3 = (y^3+3y)^2$$



                      given the Pell equation $x^2-3y^2 =1$.






                      share|cite|improve this answer











                      $endgroup$













                      • $begingroup$
                        @AlexD $r=2$ in this question.
                        $endgroup$
                        – Dietrich Burde
                        Dec 29 '18 at 23:34










                      • $begingroup$
                        Thanks, @DietrichBurde. Let me try again: A solution to the problem here is a tuple of 3 numbers. But the parameterization here only relates $x$ and $y$. What about $z$?
                        $endgroup$
                        – Alex D
                        Dec 30 '18 at 11:21










                      • $begingroup$
                        @AlexD We have $X^4+Y^3=Z^2$ and $X$,$Y$,$Z$ depend on $x$ and $y$ only. So no $z$ in here, only two parameters $x,y$ here for an equation in three variables $X,Y,Z$.
                        $endgroup$
                        – Dietrich Burde
                        Dec 30 '18 at 11:47












                      • $begingroup$
                        Well, @DietrichBurde, you can find out more, but how can you find a general formula (solve this equation in general form), expressed in arbitrary numbers (for example) m, n, k? For example, find the top three numbers (6, 5, 29)?
                        $endgroup$
                        – Yan Dashkow
                        Dec 30 '18 at 11:55










                      • $begingroup$
                        @YanDashkow We find them using one of the parametrizations, see Beukers article.
                        $endgroup$
                        – Dietrich Burde
                        Dec 30 '18 at 13:31
















                      3












                      3








                      3





                      $begingroup$

                      Here is one simple parameterization. We have,



                      $$x^4 +(y^2-1)^3 = (y^3+3y)^2$$



                      given the Pell equation $x^2-3y^2 =1$.






                      share|cite|improve this answer











                      $endgroup$



                      Here is one simple parameterization. We have,



                      $$x^4 +(y^2-1)^3 = (y^3+3y)^2$$



                      given the Pell equation $x^2-3y^2 =1$.







                      share|cite|improve this answer














                      share|cite|improve this answer



                      share|cite|improve this answer








                      edited Dec 30 '18 at 2:21

























                      answered Dec 29 '18 at 14:03









                      Tito Piezas IIITito Piezas III

                      26.9k365169




                      26.9k365169












                      • $begingroup$
                        @AlexD $r=2$ in this question.
                        $endgroup$
                        – Dietrich Burde
                        Dec 29 '18 at 23:34










                      • $begingroup$
                        Thanks, @DietrichBurde. Let me try again: A solution to the problem here is a tuple of 3 numbers. But the parameterization here only relates $x$ and $y$. What about $z$?
                        $endgroup$
                        – Alex D
                        Dec 30 '18 at 11:21










                      • $begingroup$
                        @AlexD We have $X^4+Y^3=Z^2$ and $X$,$Y$,$Z$ depend on $x$ and $y$ only. So no $z$ in here, only two parameters $x,y$ here for an equation in three variables $X,Y,Z$.
                        $endgroup$
                        – Dietrich Burde
                        Dec 30 '18 at 11:47












                      • $begingroup$
                        Well, @DietrichBurde, you can find out more, but how can you find a general formula (solve this equation in general form), expressed in arbitrary numbers (for example) m, n, k? For example, find the top three numbers (6, 5, 29)?
                        $endgroup$
                        – Yan Dashkow
                        Dec 30 '18 at 11:55










                      • $begingroup$
                        @YanDashkow We find them using one of the parametrizations, see Beukers article.
                        $endgroup$
                        – Dietrich Burde
                        Dec 30 '18 at 13:31




















                      • $begingroup$
                        @AlexD $r=2$ in this question.
                        $endgroup$
                        – Dietrich Burde
                        Dec 29 '18 at 23:34










                      • $begingroup$
                        Thanks, @DietrichBurde. Let me try again: A solution to the problem here is a tuple of 3 numbers. But the parameterization here only relates $x$ and $y$. What about $z$?
                        $endgroup$
                        – Alex D
                        Dec 30 '18 at 11:21










                      • $begingroup$
                        @AlexD We have $X^4+Y^3=Z^2$ and $X$,$Y$,$Z$ depend on $x$ and $y$ only. So no $z$ in here, only two parameters $x,y$ here for an equation in three variables $X,Y,Z$.
                        $endgroup$
                        – Dietrich Burde
                        Dec 30 '18 at 11:47












                      • $begingroup$
                        Well, @DietrichBurde, you can find out more, but how can you find a general formula (solve this equation in general form), expressed in arbitrary numbers (for example) m, n, k? For example, find the top three numbers (6, 5, 29)?
                        $endgroup$
                        – Yan Dashkow
                        Dec 30 '18 at 11:55










                      • $begingroup$
                        @YanDashkow We find them using one of the parametrizations, see Beukers article.
                        $endgroup$
                        – Dietrich Burde
                        Dec 30 '18 at 13:31


















                      $begingroup$
                      @AlexD $r=2$ in this question.
                      $endgroup$
                      – Dietrich Burde
                      Dec 29 '18 at 23:34




                      $begingroup$
                      @AlexD $r=2$ in this question.
                      $endgroup$
                      – Dietrich Burde
                      Dec 29 '18 at 23:34












                      $begingroup$
                      Thanks, @DietrichBurde. Let me try again: A solution to the problem here is a tuple of 3 numbers. But the parameterization here only relates $x$ and $y$. What about $z$?
                      $endgroup$
                      – Alex D
                      Dec 30 '18 at 11:21




                      $begingroup$
                      Thanks, @DietrichBurde. Let me try again: A solution to the problem here is a tuple of 3 numbers. But the parameterization here only relates $x$ and $y$. What about $z$?
                      $endgroup$
                      – Alex D
                      Dec 30 '18 at 11:21












                      $begingroup$
                      @AlexD We have $X^4+Y^3=Z^2$ and $X$,$Y$,$Z$ depend on $x$ and $y$ only. So no $z$ in here, only two parameters $x,y$ here for an equation in three variables $X,Y,Z$.
                      $endgroup$
                      – Dietrich Burde
                      Dec 30 '18 at 11:47






                      $begingroup$
                      @AlexD We have $X^4+Y^3=Z^2$ and $X$,$Y$,$Z$ depend on $x$ and $y$ only. So no $z$ in here, only two parameters $x,y$ here for an equation in three variables $X,Y,Z$.
                      $endgroup$
                      – Dietrich Burde
                      Dec 30 '18 at 11:47














                      $begingroup$
                      Well, @DietrichBurde, you can find out more, but how can you find a general formula (solve this equation in general form), expressed in arbitrary numbers (for example) m, n, k? For example, find the top three numbers (6, 5, 29)?
                      $endgroup$
                      – Yan Dashkow
                      Dec 30 '18 at 11:55




                      $begingroup$
                      Well, @DietrichBurde, you can find out more, but how can you find a general formula (solve this equation in general form), expressed in arbitrary numbers (for example) m, n, k? For example, find the top three numbers (6, 5, 29)?
                      $endgroup$
                      – Yan Dashkow
                      Dec 30 '18 at 11:55












                      $begingroup$
                      @YanDashkow We find them using one of the parametrizations, see Beukers article.
                      $endgroup$
                      – Dietrich Burde
                      Dec 30 '18 at 13:31






                      $begingroup$
                      @YanDashkow We find them using one of the parametrizations, see Beukers article.
                      $endgroup$
                      – Dietrich Burde
                      Dec 30 '18 at 13:31













                      0












                      $begingroup$

                      Above equation shown below has parameterization:



                      $x^3+y^4=z^2$



                      The below parameterization has no restriction such as the



                      Pell equation condition demonstrated by Tito Piezas.



                      $x=(p)^2(-q)^3$



                      $y=(p)(q)^2(k-1)$



                      $z=(p)^2(q)^4(2k-3)$



                      where, $p=(k-2)$ and $q=(k^2-2)$



                      For $k=3$ we get :
                      $(-343)^3+(98)^4=(7203)^2$






                      share|cite|improve this answer









                      $endgroup$













                      • $begingroup$
                        I have a question: through which k you can find three solutions (x, y, z) — (6,5,29)?
                        $endgroup$
                        – Yan Dashkow
                        Dec 30 '18 at 10:55


















                      0












                      $begingroup$

                      Above equation shown below has parameterization:



                      $x^3+y^4=z^2$



                      The below parameterization has no restriction such as the



                      Pell equation condition demonstrated by Tito Piezas.



                      $x=(p)^2(-q)^3$



                      $y=(p)(q)^2(k-1)$



                      $z=(p)^2(q)^4(2k-3)$



                      where, $p=(k-2)$ and $q=(k^2-2)$



                      For $k=3$ we get :
                      $(-343)^3+(98)^4=(7203)^2$






                      share|cite|improve this answer









                      $endgroup$













                      • $begingroup$
                        I have a question: through which k you can find three solutions (x, y, z) — (6,5,29)?
                        $endgroup$
                        – Yan Dashkow
                        Dec 30 '18 at 10:55
















                      0












                      0








                      0





                      $begingroup$

                      Above equation shown below has parameterization:



                      $x^3+y^4=z^2$



                      The below parameterization has no restriction such as the



                      Pell equation condition demonstrated by Tito Piezas.



                      $x=(p)^2(-q)^3$



                      $y=(p)(q)^2(k-1)$



                      $z=(p)^2(q)^4(2k-3)$



                      where, $p=(k-2)$ and $q=(k^2-2)$



                      For $k=3$ we get :
                      $(-343)^3+(98)^4=(7203)^2$






                      share|cite|improve this answer









                      $endgroup$



                      Above equation shown below has parameterization:



                      $x^3+y^4=z^2$



                      The below parameterization has no restriction such as the



                      Pell equation condition demonstrated by Tito Piezas.



                      $x=(p)^2(-q)^3$



                      $y=(p)(q)^2(k-1)$



                      $z=(p)^2(q)^4(2k-3)$



                      where, $p=(k-2)$ and $q=(k^2-2)$



                      For $k=3$ we get :
                      $(-343)^3+(98)^4=(7203)^2$







                      share|cite|improve this answer












                      share|cite|improve this answer



                      share|cite|improve this answer










                      answered Dec 30 '18 at 8:19









                      SamSam

                      1




                      1












                      • $begingroup$
                        I have a question: through which k you can find three solutions (x, y, z) — (6,5,29)?
                        $endgroup$
                        – Yan Dashkow
                        Dec 30 '18 at 10:55




















                      • $begingroup$
                        I have a question: through which k you can find three solutions (x, y, z) — (6,5,29)?
                        $endgroup$
                        – Yan Dashkow
                        Dec 30 '18 at 10:55


















                      $begingroup$
                      I have a question: through which k you can find three solutions (x, y, z) — (6,5,29)?
                      $endgroup$
                      – Yan Dashkow
                      Dec 30 '18 at 10:55






                      $begingroup$
                      I have a question: through which k you can find three solutions (x, y, z) — (6,5,29)?
                      $endgroup$
                      – Yan Dashkow
                      Dec 30 '18 at 10:55













                      0












                      $begingroup$

                      "OP" asked for parametric solution for $(x,y,z)=(6,5,29)$ in $x^3+y^4=z^2$



                      Solution is:



                      $x=3p^3(8k^2-40k+50)$



                      $y=p^2(20k^2-104k+135)$



                      $z=p^4(2k-5)^2(116k^2-540k+621)$



                      Where, $p=(4k^2-27)$



                      For $k=(13/5)$, we get after removing common factors:



                      $6^3+5^4=29^2$






                      share|cite|improve this answer









                      $endgroup$













                      • $begingroup$
                        Wait, why can't k be whole?
                        $endgroup$
                        – Yan Dashkow
                        Dec 30 '18 at 19:30
















                      0












                      $begingroup$

                      "OP" asked for parametric solution for $(x,y,z)=(6,5,29)$ in $x^3+y^4=z^2$



                      Solution is:



                      $x=3p^3(8k^2-40k+50)$



                      $y=p^2(20k^2-104k+135)$



                      $z=p^4(2k-5)^2(116k^2-540k+621)$



                      Where, $p=(4k^2-27)$



                      For $k=(13/5)$, we get after removing common factors:



                      $6^3+5^4=29^2$






                      share|cite|improve this answer









                      $endgroup$













                      • $begingroup$
                        Wait, why can't k be whole?
                        $endgroup$
                        – Yan Dashkow
                        Dec 30 '18 at 19:30














                      0












                      0








                      0





                      $begingroup$

                      "OP" asked for parametric solution for $(x,y,z)=(6,5,29)$ in $x^3+y^4=z^2$



                      Solution is:



                      $x=3p^3(8k^2-40k+50)$



                      $y=p^2(20k^2-104k+135)$



                      $z=p^4(2k-5)^2(116k^2-540k+621)$



                      Where, $p=(4k^2-27)$



                      For $k=(13/5)$, we get after removing common factors:



                      $6^3+5^4=29^2$






                      share|cite|improve this answer









                      $endgroup$



                      "OP" asked for parametric solution for $(x,y,z)=(6,5,29)$ in $x^3+y^4=z^2$



                      Solution is:



                      $x=3p^3(8k^2-40k+50)$



                      $y=p^2(20k^2-104k+135)$



                      $z=p^4(2k-5)^2(116k^2-540k+621)$



                      Where, $p=(4k^2-27)$



                      For $k=(13/5)$, we get after removing common factors:



                      $6^3+5^4=29^2$







                      share|cite|improve this answer












                      share|cite|improve this answer



                      share|cite|improve this answer










                      answered Dec 30 '18 at 18:09









                      SamSam

                      1




                      1












                      • $begingroup$
                        Wait, why can't k be whole?
                        $endgroup$
                        – Yan Dashkow
                        Dec 30 '18 at 19:30


















                      • $begingroup$
                        Wait, why can't k be whole?
                        $endgroup$
                        – Yan Dashkow
                        Dec 30 '18 at 19:30
















                      $begingroup$
                      Wait, why can't k be whole?
                      $endgroup$
                      – Yan Dashkow
                      Dec 30 '18 at 19:30




                      $begingroup$
                      Wait, why can't k be whole?
                      $endgroup$
                      – Yan Dashkow
                      Dec 30 '18 at 19:30











                      0












                      $begingroup$

                      "OP" enquired about integer coefficent's for the parametric



                      solution for the equation $(x^2+y^4=z^2)$. "OP" just needs



                      to substitute $k=(m/n)$ in the parametrization & the resulting



                      solution after removing common factors is given below.



                      $x=6(u^3)(v^2)$



                      $y=(u^2)(v)(10m-27n)$



                      $z=(u^4)(v^2)(116m^2-540mn+621n^2)$



                      And $u=(4m^2-27n^2)$ & $v=(2m-5n)$



                      For $(m,n)=(13,5)$ we get:



                      $6^3+5^4=29^2$






                      share|cite|improve this answer









                      $endgroup$


















                        0












                        $begingroup$

                        "OP" enquired about integer coefficent's for the parametric



                        solution for the equation $(x^2+y^4=z^2)$. "OP" just needs



                        to substitute $k=(m/n)$ in the parametrization & the resulting



                        solution after removing common factors is given below.



                        $x=6(u^3)(v^2)$



                        $y=(u^2)(v)(10m-27n)$



                        $z=(u^4)(v^2)(116m^2-540mn+621n^2)$



                        And $u=(4m^2-27n^2)$ & $v=(2m-5n)$



                        For $(m,n)=(13,5)$ we get:



                        $6^3+5^4=29^2$






                        share|cite|improve this answer









                        $endgroup$
















                          0












                          0








                          0





                          $begingroup$

                          "OP" enquired about integer coefficent's for the parametric



                          solution for the equation $(x^2+y^4=z^2)$. "OP" just needs



                          to substitute $k=(m/n)$ in the parametrization & the resulting



                          solution after removing common factors is given below.



                          $x=6(u^3)(v^2)$



                          $y=(u^2)(v)(10m-27n)$



                          $z=(u^4)(v^2)(116m^2-540mn+621n^2)$



                          And $u=(4m^2-27n^2)$ & $v=(2m-5n)$



                          For $(m,n)=(13,5)$ we get:



                          $6^3+5^4=29^2$






                          share|cite|improve this answer









                          $endgroup$



                          "OP" enquired about integer coefficent's for the parametric



                          solution for the equation $(x^2+y^4=z^2)$. "OP" just needs



                          to substitute $k=(m/n)$ in the parametrization & the resulting



                          solution after removing common factors is given below.



                          $x=6(u^3)(v^2)$



                          $y=(u^2)(v)(10m-27n)$



                          $z=(u^4)(v^2)(116m^2-540mn+621n^2)$



                          And $u=(4m^2-27n^2)$ & $v=(2m-5n)$



                          For $(m,n)=(13,5)$ we get:



                          $6^3+5^4=29^2$







                          share|cite|improve this answer












                          share|cite|improve this answer



                          share|cite|improve this answer










                          answered Dec 31 '18 at 11:49









                          SamSam

                          1




                          1






























                              draft saved

                              draft discarded




















































                              Thanks for contributing an answer to Mathematics Stack Exchange!


                              • Please be sure to answer the question. Provide details and share your research!

                              But avoid



                              • Asking for help, clarification, or responding to other answers.

                              • Making statements based on opinion; back them up with references or personal experience.


                              Use MathJax to format equations. MathJax reference.


                              To learn more, see our tips on writing great answers.




                              draft saved


                              draft discarded














                              StackExchange.ready(
                              function () {
                              StackExchange.openid.initPostLogin('.new-post-login', 'https%3a%2f%2fmath.stackexchange.com%2fquestions%2f3055812%2fhow-to-solve-this-diophantine-equation%23new-answer', 'question_page');
                              }
                              );

                              Post as a guest















                              Required, but never shown





















































                              Required, but never shown














                              Required, but never shown












                              Required, but never shown







                              Required, but never shown

































                              Required, but never shown














                              Required, but never shown












                              Required, but never shown







                              Required, but never shown







                              Popular posts from this blog

                              How do I know what Microsoft account the skydrive app is syncing to?

                              When does type information flow backwards in C++?

                              Grease: Live!